LSAT and Law School Admissions Forum

Get expert LSAT preparation and law school admissions advice from PowerScore Test Preparation.

User avatar
 Dave Killoran
PowerScore Staff
  • PowerScore Staff
  • Posts: 5850
  • Joined: Mar 25, 2011
|
#27180
Complete Question Explanation

Flaw in the Reasoning. The correct answer choice is (B)

This is a difficult question for many students, primarily because of the wording of the correct answer choice.

Jane's argument is straightforward but has a very absolute conclusion: Because hand-eye coordination suffers when a lot of tv is watched, Jacqueline and Mildred must have their tv time restricted.

Alan’s argument is that because the study showing that television watching is harmful was specific to children under three, and since Jacqueline and Mildred are over three, then the results of the study do not apply to them. Of course, the flaw here is that even though Jacqueline and Mildred are older than the children in the study, it is still entirely possible that watching television could also have some harmful effects on them as well. On this basis, Alan makes a conclusion equally absolute, but in the other direction: "we need not restrict their television viewing." But has Alan proven this point?

The answer is No. What Alan has done is show that Jane's argument isn't proven and that her conclusion is weakened. Thus, Alan's conclusion should be that we don't necessarily need to restrict the amount of time they watch tv. But, instead Alan went full force and concluded something far more certain than what he had proven. Now you must find an answer that shows how he overreached. Note: this is a classic error in the use of evidence and can be classified in the "Some evidence against a position is taken to prove that the position is false" category. This form has appeared on the LSAT multiple times, and this is a question you should study closely to understand the mechanics of the concept.


Answer choice (A): While this is true of Alan’s argument, it is not a flaw in his argument. Concluding something from the same source is fine; it is the force of his conclusion that is flawed.

Answer choice (B): This is the correct answer choice. Essentially what Alan does is make an absolute conclusion (we need not restrict TV viewing) without actually proving that Jane’s argument is completely wrong. He weakens her argument a bit by showing that the study may not apply to J and M (undermining her premise in support of her conclusion), but he does not prove that television is not in some way (the same as Jane mentions or otherwise) harmful, which is what Jane concludes. Remember, to show that something needs (or must) or does not need to (or cannot) happen you must prove your point beyond any doubt.

Answer choice (C): Alan’s argument focuses on Jane’s premise, which is certainly not a side issue.

Answer choice (D): This is an appeal to authority flaw and does not appear in Alan’s argument.

Answer choice (E): The practice in question is watching television, and while the consequences (negative effects) are being debated, the causes of the practice of watching television are never addressed. Hence this answer choice does not apply.
 reop6780
  • Posts: 265
  • Joined: Jul 27, 2013
|
#12748
The correct answer is B but i chose C.

The expression of "confuses" is what hindered me from considering answer B.

I remember one of the replies that i got regarding flaw types mentioned that certain answer was wrong because there is now confusion in the stimuli.

How do I do whether Alan was confused when i do not know what her original intention was?

- was she trying to disprove the conclusion made by Jane?

- how is undermining an argument so much different from disproving the conclusion?

- Is Alan in any support of a given conclusion? ; I dont see it in the stimuli.
Regarding the incorrect answer C, I thought the main point of Jane was addressing the danger of watching tv, and Alan merely focused on the age matter.
What mistake am i making in answer C?
 Lucas Moreau
PowerScore Staff
  • PowerScore Staff
  • Posts: 216
  • Joined: Dec 13, 2012
|
#12818
Hello, reop,

The trick is whether Alan is attacking the conclusion itself or merely attacking the evidence advanced in support of that conclusion. In this case, Alan is doing the second, but thinks he is doing the first.

Jane says that, because of the article (the evidence), they should restrict their children's television viewing. Alan says that the article does not apply to their children (attacks the evidence).

Where his flaw lies is thinking that just because the article doesn't apply, Jane's conclusion (that they should restrict their children's television viewing) is completely disproven. It isn't! There might be some other reason that Jane and Alan should restrict their children's television viewing than the reason advanced in the article. (Perhaps they're watching too much TV and not studying enough...? :ras: )

Basically, just because the conclusion is shown to not be proven by the evidence is not conclusive disproof of that conclusion. As such, attacking the evidence - even successfully - isn't the same as showing the conclusion itself is false. Hence C is the best answer.

Hope that helps,
Lucas Moreau
PowerScore
 reop6780
  • Posts: 265
  • Joined: Jul 27, 2013
|
#13179
Thank you !! It helped me a lot!
 FIDELIO
  • Posts: 12
  • Joined: Sep 19, 2014
|
#17052
Really confused here, please help... :-? :-? :-?

I first tried to solve this problem without conditional reasoning.

The flaw I prephrased in Alan's argument was something along the lines of: perhaps there are other reasons why we should restrict kids TV time other than it affecting their hand-eye coordination. Perhaps, despite the kids age, watching to much TV could lead other disastrous affects aside from those mentioned in the article Jane was referring to.

So based on this information I went though the answer choices and was left with B and E as possible contenders. I could not make out IN LAYMAN'S TERMS terms what each of them were saying, so I broke out conditional reasoning.

JANE:

HECS :arrow: NO TV


ALAN:

NO HECS :arrow: TV
contra: NO TV :arrow: HECS

so it looks like Alan has committed the error of mistaken reversal. But isn't the error of mistaken reversal the same thing as the mistaken negation. That is to say the contrapostive of an error of mistaken reversal is the mistaken negation. Which is why I went with E. Doesn't E describe the mistaken reversal flaw shown above?

The OA says B.

Can someone help me understand answer choices B and E.


:-? :-? :-?
 BethRibet
PowerScore Staff
  • PowerScore Staff
  • Posts: 200
  • Joined: Oct 17, 2012
|
#17054
Hi Fidelio,

Thanks for the question. I believe I follow your logic, but this argument is not actually based on conditional reasoning. Alan is making an error in the use of evidence.

In the simplest terms, the idea is that a some evidence against a position (or a lack of evidence for it), does not make the opposite true. That is: If someone tells you, "prove you will have a good day tomorrow", and you say, "I can't", that doesn't necessarily mean we can conclude that you will certainly not have a good day. It would be flawed to conclude: "Therefore, you will have a bad day tomorrow.

Or for instance:
I said, "We could spend a million dollars today."
And you answered: "We don't have a million dollars, so we should not spend a million dollars."
And I whipped out a bank statement and said, "We do, you're wrong, so we should spend it."

The problem with my logic is that just because I've struck down your objection doesn't mean I've adequately supported my conclusion -- that we should spend the money. That conclusion requires more support, or evidence, like why we can spare that much, and what good use we would put it to if we suddenly spent it, or perhaps a principle about when we should spend that much money, such as "When we have a million dollars, it should be spent within 24 hours."

Alan is doing something similar -- he's struck down Jane's evidence supporting her conclusion (her conclusion = restrict TV watching), and because of that, he concludes that the opposite is true: don't restrict TV watching. But there might be other evidence or reasons to restrict TV watching. We don't have enough information to support his conclusion.

Answer choice B describes this flaw: Alan undermined Jane's argument, but that doesn't mean he definitely proved Jane wrong.

Hope this helps!
Beth
 FIDELIO
  • Posts: 12
  • Joined: Sep 19, 2014
|
#17055
BethRibet wrote: The problem with my logic is that just because I've struck down your objection doesn't mean I've adequately supported my conclusion
(B) It confuses undermining an argument in
support of a given conclusion with showing
that the conclusion itself is false.

Thanks for the help. I wish they would have put the correct answer using your words above.

This was so helpful.

Thank you. :-D :-D :-D
 Mustafaabdulmalek
  • Posts: 19
  • Joined: Nov 17, 2015
|
#21398
my problem with this question is that I do not see the flaw in Alan argument since the source is the article for both side and the article said that ONLY children under 3 years so why he can not conclude that Jane is wrong ,also do we have or can we assume that just because the article said that that does not mean that there no other explanation, I mean how should I treat this question ? like a must be true or can I assume something from outside ?
if we decide the "TV watching" just by considering the article why is it flawed ?
thanks
 Adam Tyson
PowerScore Staff
  • PowerScore Staff
  • Posts: 5153
  • Joined: Apr 14, 2011
|
#21402
Since this is a Flaw in the Reasoning question, we don't really want any outside information - rather, we want to explain what it was Alan did wrong. If you see no flaw in Alan's argument then you must agree with his conclusion, right? Do you agree that Jane's argument is "rubbish" and that there is absolutely no reason to restrict how much TV the kids watch?

What Alan did was pretty extreme. He did a good job of pointing out that Jane's argument was built on evidence that was not (entirely) applicable to her conclusion - their kids are older than the ones studied. Alan could rightfully be said to have weakened Jane's argument. But as you know from doing Weaken questions in LR, it's a long way from "weaken" to "rubbish!" Weaken answers do some damage, certainly, but they don't need to destroy the argument completely (and they rarely are that conclusive). There's the problem for Alan - he weakened, but then he went overboard and presumed that he had destroyed Jane's argument. What if there are other reasons to restrict TV viewing? What if the article is wrong and it also affects older kids in the same way?

Try taking the argument personally - put yourself in the shoes of each of the speakers. If you were Alan, would you feel that you had done such a great job that you could "drop the mic" and walk out victorious? If you were Jane, would you say "gee, I guess you're right, what an idiot I am" or would you perhaps have some sort of comeback for Alan? Taking the stimulus personally, getting into a debate with the authors in your mind, will better help you to pick arguments apart, weaken them, strengthen them, etc.

Take another look and give that some thought, especially about how Jane might reply to Alan.

Keep up the good work!
 Mustafaabdulmalek
  • Posts: 19
  • Joined: Nov 17, 2015
|
#21403
we don't really want any outside information
What if there are other reasons to restrict TV viewing? What if the article is wrong and it also affects older kids in the same way?
This what confuses me the "what if" can I think this way
And not going out the stimuli ??
Thanks

Get the most out of your LSAT Prep Plus subscription.

Analyze and track your performance with our Testing and Analytics Package.